Can someone please tell me how to solve this problem??!! I literally have to go back in math if I don’t pass this HELP!!

Can Someone Please Tell Me How To Solve This Problem??!! I Literally Have To Go Back In Math If I Dont

Answers

Answer 1

Answer:

          D.   270° < φ < 360°

Step-by-step explanation:

Imagine coordinate system

I quarter is where x>0 and y>0  {right top} and it is (0°,90°)

II quarter is where x<0 and y>0  {left top} and it is (90°,180°)

III quarter is where x<0 and y<0  {left bottom} and it is (180°,270°)

IV quarter is where x>0 and y<0  {right bottom} and it is (270°,360°)

Now, we have an angle wich vertex is point (0,0) and one of its sides is X-axis and the second lay at one of the quarters.

For the trig functons of an angle created by this second side always are true:

In first quarter all functions are >0

in second one only sine

in third one: tangent and cotangent

and in fourth one: cosine

{You can check this by selecting any point on the second side of angle and put it's coordinates to formulas of these functions:

[tex]\sin \phi=\dfrac y{\sqrt{x^2+y^2}}\,,\quad \cos \phi=\dfrac x{\sqrt{x^2+y^2}}\,,\quad \tan\phi=\dfrac yx\,,\quad \cot\phi=\dfrac xy[/tex]  }

So:

sinφ<0  ⇒ III or IV quarter

tanφ<0  ⇒ I or IV quarter

IV quarter  ⇒  φ ∈ (270°, 360°)


Related Questions

Brian invested his savings in two investment funds. The $8000 that he invested in Fund A returned a 4% profit. The amount that he invested in Fund B returned a 1% profit. How much did he invest in Fund B, if both funds together returned a 2% profit?

Answers

Answer: Brian invested $16000 in Fund B .

Step-by-step explanation:

Let x be the amount Brian invested in Fund B.

Given, The $8000 that he invested in Fund A returned a 4% profit. The amount that he invested in Fund B returned a 1% profit.

i.e. profit on Fund A = 4% of 8000 = 0.04 ×8000 = $320

Profit on Fund B = 1% of x = 0.01x

Together they earn 1% profit, i.e. Combined profit = 2% of (8000+x)

= 0.02(8000+x)

As per question,

Combined profit=Profit on Fund A+Profit on Fund B

[tex]\Rightarrow\ 0.02(8000+x) =320+0.01x\\\\\Rightarrow\ 0.02(8000) +0.02x=320+0.01x\\\\\Rightarrow\ 160+0.02x=320+0.01x\\\\\Rightarrow\ 0.02x-0.01x=320-160\\\\\Rightarrow\ 0.01x=160\\\\\Rightarrow\ x=\dfrac{160}{0.01}\\\\\Rightarrow\ x=16000[/tex]

Hence, Brian invested $16000 in Fund B .

(08.02)How many solutions are there for the system of equations shown on the graph? No solution One solution Two solutions Infinitely many solutions

Answers

Answer: Infinitely many solutions

Step-by-step explanation:

There are many solutions because the lines lies on top of each other.

i dont know the exact answer but its not  

One solution

Two solutions

so its most likely

Infinitely many solutions

Kelsey had $65 to spend on books. Each book cost $5.50, and there was a $7.50 fee for shipping. She let b equal the number of books she can purchase and wrote the inequality 5.50 b + 7.5 less-than 65 to represent the situation. Which statements describe the reasoning used to determine if Kelsey’s inequality is correct? Select two options. The inequality symbol is correct because she must spend less than $65. The inequality symbol is incorrect because she can spend up to and including $65. The expression 5.50b + 7.5 is correct because $5.50 per book is 5.50b and that is added to the shipping fee of $7.50 to determine the total purchase price. The expression 5.50b + 7.5 is incorrect because $5.50 per book and $7.50 should be combined to $9.50b to determine the total purchase price. The inequality symbol is correct because she cannot spend more than $65.

Answers

The statements that can be used to describe the reasoning used to determine if Kelsey’s inequality is correct include:

The inequality symbol is incorrect because she can spend up to and including $65. The expression 5.50b + 7.5 is correct because $5.50 per book is 5.50b and that is added to the shipping fee of $7.50 to determine the total purchase price.

It should be noted that the inequality symbol is incorrect because she can spend up to and including $65.

Based on the information given, the correct expression that can be used to solve the question should be:

65 - (5.50b + 7.5)

In conclusion, the correct options are B and C.

Read related link on:

https://brainly.com/question/16904821

Answer:

B and C

Step-by-step explanation:

What is the solution to 7 × p = -56? A. -49 B. -8 C. 8 D. 49

Answers

Answer:

-8

Step-by-step explanation:

Hello!

What we do to one side of the equation we do to the other side

7 * p = -56

Divide both sides by 7

p = -8

The answer is -8

Hope this helps!

what do you think you’d like most about working as a forensic scientist? why

Answers

Answer:

i think its very interesting and pretty cool,  because there is so much to learn  and so much to explore

i wouldn't like the fact that you have to study so much though

Step-by-step explanation:

Represents the solution to the inequality -9=2/3x-7<5

Answers

Answer:

-3=x <13

Step-by-step explanation:

[tex] - 9 = \frac{2x}{3} - 7 < 5[/tex]

Multiply through by 3

[tex] - 27 = 2x - 21 < 15[/tex]

Add 21 to all sides

[tex] - 6 = 2x < 36[/tex]

Divide through by 2

[tex] - 3 = x < 18[/tex]

The solutin set is

[tex]{- 3 = x < 18}[/tex]

Which equation represents the line that is perpendicular to y=3/4x+1 and passes through (-5,11)
Will give brainliest!!

Answers

Answer:

y = - [tex]\frac{4}{3}[/tex] x + [tex]\frac{13}{3}[/tex]

Step-by-step explanation:

The equation of a line in slope- intercept form is

y = mx + c ( m is the slope and c the y- intercept )

y = [tex]\frac{3}{4}[/tex] x + 1 ← is in slope- intercept form

with slope m = [tex]\frac{3}{4}[/tex]

Given a line with slope m then the slope of a line perpendicular to it is

[tex]m_{perpendicular}[/tex] = - [tex]\frac{1}{m}[/tex] = - [tex]\frac{1}{\frac{3}{4} }[/tex] = - [tex]\frac{4}{3}[/tex] , thus

y = - [tex]\frac{4}{3}[/tex] x + c ← is the partial equation

To find c substitute (- 5, 11) into the partial equation

11 = [tex]\frac{20}{3}[/tex] + c ⇒ c = 11 - [tex]\frac{20}{3}[/tex] = [tex]\frac{13}{3}[/tex]

y = - [tex]\frac{4}{3}[/tex] x + [tex]\frac{13}{3}[/tex] ← equation of perpendicular line

The equation of the line that passes through (-5, 11) and perpendicular to y = (3/4)x + 1 is

y = -2x + 1

What is an equation of a line?

The equation of a line is given by:

y = mx + c

where m is the slope of the line and c is the y-intercept.

Example:

The slope of the line y = 2x + 3 is 2.

The slope of a line that passes through (1, 2) and (2, 3) is 1.

We have,

y = (2/4)x + 1 is in the form of y = m(2)x + c

So,

m(2) = 2/4 = 1/2

The equation of the line y = m(1)x + c is perpendicular to y = (2/4)x + 1.

So,

m(1) x m(2) = -1

m(1) = -1/(1/2)

m(1) = -2

Now,

y = -2x + c passes through (-5, 11).

This means,

11 = -2 x (-5) + c

11 = 10 + c

11- 10 = c

c = 1

Thus,

The equation of the line is y = -2x + 1.

Learn more about equation of a line here:

https://brainly.com/question/23087740

#SPJ2

how do you solve 2m-10=44+8m

Answers

Answer:

m = -9

Step-by-step explanation:

2m-10=44+8m

Subtract 2m from each side

2m-2m-10=44+8m-2m

-10 = 44+6m

Subtract 44 from each side

-10-44 = 44-44+6m

-54 = 6m

Divide by 6

-54/6 = 6m/6

-9 = m

Answer:

solve by solving the salvation for equation don't be a slave get educated from what's gave

Complete the square to transform the expression x2 - 2x - 2 into the form a(x - h)2 + k

Answers

Answer:

A

Step-by-step explanation:

Find the vertex form of the quadratic function below.

y = x^2 - 4x + 3

This quadratic equation is in the form y = a{x^2} + bx + cy=ax  

2

+bx+c. However, I need to rewrite it using some algebraic steps in order to make it look like this…

y = a(x - h)^2 + k

This is the vertex form of the quadratic function where \left( {h,k} \right)(h,k) is the vertex or the “center” of the quadratic function or the parabola.

Before I start, I realize that a = 1a=1. Therefore, I can immediately apply the “completing the square” steps.

STEP 1: Identify the coefficient of the linear term of the quadratic function. That is the number attached to the xx-term.

STEP 2: I will take that number, divide it by 22 and square it (or raise to the power 22).

STEP 3: The output in step #2 will be added and subtracted on the same side of the equation to keep it balanced.

Think About It: If I add 44 on the right side of the equation, then I am technically changing the original meaning of the equation. So to keep it unchanged, I must subtract the same value that I added on the same side of the equation.

STEP 4: Now, express the trinomial inside the parenthesis as a square of a binomial, and simplify the outside constants.

After simplifying, it is now in the vertex form y = a{\left( {x - h} \right)^2} + ky=a(x−h)  

2

+k where the vertex \left( {h,k} \right)(h,k) is \left( {2, - 1} \right)(2,−1).

Visually, the graph of this quadratic function is a parabola with a minimum at the point \left( {2, - 1} \right)(2,−1). Since the value of “aa” is positive, a = 1a=1, then the parabola opens in upward direction.

Example 2: Find the vertex form of the quadratic function below.

The approach to this problem is slightly different because the value of “aa” does not equal to 11, a \ne 1a  

​  

=1. The first step is to factor out the coefficient 22 between the terms with xx-variables only.

STEP 1: Factor out 22 only to the terms with variable xx.

STEP 2: Identify the coefficient of the xx-term or linear term.

STEP 3: Take that number, divide it by 22, and square.

STEP 4: Now, I will take the output {9 \over 4}  

4

9

​  

 and add it inside the parenthesis.

By adding {9 \over 4}  

4

9

​  

 inside the parenthesis, I am actually adding 2\left( {{9 \over 4}} \right) = {9 \over 2}2(  

4

9

​  

)=  

2

9

​  

 to the entire equation.

Why multiply by 22 to get the “true” value added to the entire equation? Remember, I factored out 22 in the beginning. So for us to find the real value added to the entire equation, we need to multiply the number added inside the parenthesis by the number that was factored out.

STEP 5: Since I added {9 \over 2}  

2

9

​  

 to the equation, then I should subtract the entire equation by {9 \over 2}  

2

9

​  

 also to compensate for it.

STEP 6: Finally, express the trinomial inside the parenthesis as the square of binomial and then simplify the outside constants. Be careful combining the fractions.

It is now in the vertex form y = a{\left( {x - h} \right)^2} + ky=a(x−h)  

2

+k where the vertex \left( {h,k} \right)(h,k) is \left( {{{ - \,3} \over 2},{{ - 11} \over 2}} \right)(  

2

−3

​  

,  

2

−11

​  

).

Example 3: Find the vertex form of the quadratic function below.

Solution:

Factor out - \,3−3 among the xx-terms.

The coefficient of the linear term inside the parenthesis is - \,1−1. Divide it by 22 and square it. Add that value inside the parenthesis. Now, figure out how to make the original equation the same. Since we added {1 \over 4}  

4

1

​  

 inside the parenthesis and we factored out - \,3−3 in the beginning, that means - \,3\left( {{1 \over 4}} \right) = {{ - \,3} \over 4}−3(  

4

1

​  

)=  

4

−3

​  

 is the value that we subtracted from the entire equation. To compensate, we must add {3 \over 4}  

4

3

​  

 outside the parenthesis.

Therefore, the vertex \left( {h,k} \right)(h,k) is \left( {{1 \over 2},{{11} \over 4}} \right)(  

2

1

​  

,  

4

11

​  

).

Example 4: Find the vertex form of the quadratic function below.

y = 5x^2 + 15x - 5  

Solution:

Factor out 55 among the xx-terms. Identify the coefficient of the linear term inside the parenthesis which is 33. Divide it by 22 and square to get {9 \over 4}  

4

9

​  

.

Add {9 \over 4}  

4

9

​  

 inside the parenthesis. Since we factored out 55 in the first step, that means 5\left( {{9 \over 4}} \right) = {{45} \over 4}5(  

4

9

​  

)=  

4

45

​  

 is the number that we need to subtract to keep the equation unchanged.

Express the trinomial as a square of binomial, and combine the constants to get the final answer.

Therefore, the vertex \left( {h,k} \right)(h,k) is {{ - \,3} \over 2},{{ - \,65} \over 4}  

2

−3

​  

,  

4

−65

​  

.

Answer:

(x - 1 )^2 - 3

Step-by-step explanation:

( x - 1 )^2 + ( -3)

x^2 - 2x + 1 - 3

x^2 - 2x - 2

4x + 5y = 19 , 5y - 4x = 38​

Answers

Answer:

Step-by-step explanation:

Adding both equations

4x+5y+5y-4x=19+38

10y = 57

y= 5.7

Subtracting equation i from ii

5y-4x-4x-5y=38-19

-8x=9

x= -0.9

math now..!! Help..?

Answers

Answer:

2p + 12

2 (p = +6) + 12 = 20

Answer:

I think its 6

Step-by-step explanation:

because you have to add 9 and 3 together then you get 12 and you have to divide 2p with 12 and you'll get 6

Find the value of x so that the function has the given value.


j(x)=−4/5x+7; j(x)=−5

x=



Answers

Answer:

x = 3

Step-by-step explanation:

j(x) = 4/5(-5) + 7

= -4 + 7

= 3

Answer:

15

Step-by-step explanation: -4/5 x has to be -12 because -12+7 equals 5. Since we want to figure out x, we have to flip -4/5 x to 4/5x which would change the -12 to 12. What is a fourth of 12? It is three. 12+3 equals 15. This is the first right answer on all of the internet for this question!

The expression 6(x − 5) means the . If x = 7, the value of the expression is

Answers

Answer:

Hey there!

6(x-5)

6(7-5)

6(2)

12

Hope this helps :)

Answer:

12

Step-by-step explanation:

Replace x by 7 in 6(x-5) to be able to evaluate the expression.

● 6(x-5)

● 6(7-5)

● 6 × 2

● 12

So the expression is equal to 12 when x=7

Pls answer I really need help
Brainlist and thank you will be the reward thank you so much!!!

Answers

Answer:

0.667 ✅

Step-by-step explanation:

This is best solved using a proportion.

The formula is soy/vinegar = soy/vinegar where one of these is a variable.

Here we have:

[tex]\frac{150}{100} = \frac{1}{x}[/tex]

Now, we solve this by cross multiplying.

150x = 100

Dividing both sides by x, we get x = 2/3 or about 0.667.

Checking:

[tex]\frac{150}{100} = \frac{1}{0.667}[/tex]

1.5 = 1.5 ✅

I'm always happy to help :)

simplify 5 x 5^2 in index form

Answers

Answer:

5x(25)

Step-by-step explanation:

NEED ASAP What is the quotient and remainder of 8,595 ÷ 24?

Answers

Answer:

358.125

Step-by-step explanation:

Answer:

358 3/24

Step-by-step explanation:  

What is the smallest positive integer $n$ such that $\frac{n}{n+101}$ is equal to a terminating decimal?

Answers

Answer:

n = 24

Step-by-step explanation:

Given the fraction:

[tex]$\frac{n}{n+101}$[/tex]

To find:

Smallest positive integer [tex]$n$[/tex] such that the fraction is equal to a terminating decimal.

Solution:

The rule that a fraction is equal to a terminating decimal states that, the denominator must contain factors of only 2 and 5.

i.e. Denominator must look like [tex]2^m\times 5^n[/tex], only then the fraction will be equal to a terminating decimal.

Now, let us have a look at the denominator, [tex]n+101[/tex]

Let us use hit and trial method to find the value of [tex]n[/tex] as positive integer.

n = 1, denominator becomes 102 = [tex]2 \times 3 \times 17[/tex] not of the form [tex]2^m\times 5^n[/tex].

n = 4, denominator becomes 105 = [tex]5 \times 3 \times 7[/tex] not of the form [tex]2^m\times 5^n[/tex].

n = 9, denominator becomes 110 = [tex]2 \times 5 \times 11[/tex] not of the form [tex]2^m\times 5^n[/tex].

n = 14, denominator becomes 115 = [tex]5 \times 23[/tex] not of the form [tex]2^m\times 5^n[/tex].

n = 19, denominator becomes 120 = [tex]5 \times 3 \times 2^3[/tex] not of the form [tex]2^m\times 5^n[/tex].

n = 24, denominator becomes 125 = [tex]2^0 \times 5 ^3[/tex] It is of the form [tex]2^m\times 5^n[/tex].

So, the answer is n = 24

Shaquira is baking cookies to put in packages for a fundraiser. Shaquira has made 86 8686 chocolate chip cookies and 42 4242 sugar cookies. Shaquira wants to create identical packages of cookies to sell, and she must use all of the cookies. What is the greatest number of identical packages that Shaquira can make?

Answers

Answer: 2

Step-by-step explanation:

Given: Shaquira has made 86  chocolate chip cookies and 42 sugar cookies.

Shaquira wants to create identical packages of cookies to sell, and she must use all of the cookies.

Now, the greatest number of identical packages that Shaquira can make= GCD of 86 and 42

Prime factorization of 86 and 42:

86 = 2 ×43

42 = 2 × 3 × 7

GCD of 86 and 42 = 2   [GCD = greatest common factor]

Hence, the greatest number of identical packages that Shaquira can make =2

Puzzle corner
Look Before You Leap!
See how long it takes you to work out the
following:
(1 x2)×(3 x 4)×(586)×(7 x 8) x (
9×0)

Answers

Answer:

0

Step-by-step explanation:

Notice that the last factor is null (9×0)

So the result will be null since any number that is multiplied by 0 equals 0.

I need Helpppp quick!!!!

Answers

Answer:

G

Step-by-step explanation:

let his fixed price be x and his hourly fee be y;

270 = 4y + x

420 = 7y + x

x is common in both equations

equate the two;

x = 270-4y and x = 420-7y

270-4y = 420-7y

3y = 150

y = 50

x = 270-4*50

x = 70

Review what you know about products and sums represented by rectangular area models. [5 points] Use algebra tiles to multiply (x-1)(3x+2).

Answers

(x - 1)(3x + 2)
= 3x^2 + 2x - 3x - 2
= 3x^2 - x - 2

3x^2 - x - 2

What are some ways to solve an equation?

Different ways to solve equations. We have 4 ways of solving one-step equations: Adding, Substracting, multiplication, and division. If we add the same number to both sides of an equation, both sides will remain equal.

How do you evaluate an equation?

∫ y2+y−2dy ∫ y 2 + y − 2 d y∫ 2 1 y2 +y−2dy ∫ 1 2 y 2 + y − 2 d y∫ 2 −1 y2 +y−2dy ∫ − 1 2 y 2 + y − 2 d y

= (x - 1)(3x + 2)

= 3x^2 + 2x - 3x - 2

= 3x^2 - x - 2

Learn more about equation here https://brainly.com/question/10413253

#SPJ2

I need helps will give you a good rating.

Answers

Answer: x = 3

Step-by-step explanation:

Sqrt(x+7) - 1  = x

Sqrt(x+7) = x + 1

x+7 = x^2 + 1

x = x^2 - 6

x=3

1. Suzette ran and biked for a total of 80 miles in 9 hours. Her average running speed was 5 miles per hour (mph) and her average biking speed was 12 mph. Let x = total hours Suzette ran. Let y = total hours Suzette biked. Use substitution to solve for x and y. Show your work. Check your solution. (a) How many hours did Suzette run? (b) How many hours did she bike?

Answers

Answer:

a) Suzette ran for 4 hours

b) Suzette biked for 5 hours

Step-by-step explanation:

Speed is rate of distance traveled, it is the ratio of distance traveled to time taken. It is given by:

Speed = distance / time

The total distance ran and biked by Suzette (d) = 80 miles, while the total time ran and biked by Suzette (t) = 9 hours.

For running:

Her speed was 5 miles per hour, let the total hours Suzette ran be x and the total distance she ran be p, hence since Speed = distance / time, therefore:

5 = p / x

p = 5x

For biking:

Her speed was 12 miles per hour, let the total hours Suzette ran be y and the total distance she ran be q, hence since Speed = distance / time, therefore:

12 = q / y

q = 12y

The total distance ran and biked by Suzette (d) = Distance biked + distance ran

d = p + q

80 = p + q

80 = 5x + 12y                 (1)

The total time taken to run and bike by Suzette (t) = time spent to bike + time spent to run

t = x + y

9 = x + y                         (2)

Solving equation 1 and equation 2, multiply equation 2 by 5 and subtract from equation 1:

7y = 35

y = 35/7

y = 5 hours

Put y = 5 in equation 2:

9 = x + 5

x = 9 -5

x = 4 hours

a) Suzette ran for 4 hours

b) Suzette biked for 5 hours

Which equation can be used to find x, the length of the hypotenuse of the right triangle?

Answers

Answer:

[tex] \boxed{\sf {18}^{2} + {24}^{2} = {x}^{2}} [/tex]

To Find:

Length of hypotenuse of the right triangle i.e. x

Step-by-step explanation:

Pythagoras theorem states that “In a right-angled triangle, the square of the hypotenuse side is equal to the sum of squares of the other two sides”.

[tex] \therefore [/tex]

[tex] \sf \implies {18}^{2} + {24}^{2} = {x}^{2} [/tex]

Answer:

18²+24²=x²

Step-by-step explanation:

to answer this question you must know Pythagorean theorem

a^ 2+b^2 =c^2

a and b stands for the sides with length 24 and 18 and c stands for the HYPOTENUSE . so the correct answer for the above question is 18²+24²=x²

PLEASE HELP
Find the area and the perimeter of the shaded regions below. Give your answer as a completely simplified exact value in terms of π (no approximations). The figures below are based on semicircles or quarter circles and problems b), c), and d) are involving portions of a square.

Answers

Answer:

perimeter is  4 sqrt(29) + 4pi  cm

area is 40 + 8pi cm^2

Step-by-step explanation:

We have a semicircle and a triangle

First the semicircle with diameter 8

A = 1/2 pi r^2 for a semicircle

r = d/2 = 8/2 =4

A = 1/2 pi ( 4)^2

  =1/2 pi *16

  = 8pi

Now the triangle with base 8 and height 10

A = 1/2 bh

  =1/2 8*10

  = 40

Add the areas together

A = 40 + 8pi cm^2

Now the perimeter

We have 1/2 of the circumference

1/2 C =1/2 pi *d

         = 1/2 pi 8

        = 4pi

Now we need to find the length of the hypotenuse of the right triangles

using the pythagorean theorem

a^2+b^2 = c^2

The base is 4 ( 1/2 of the diameter) and the height is 10

4^2 + 10 ^2 = c^2

16 + 100 = c^2

116 = c^2

sqrt(116) = c

2 sqrt(29) = c

Each hypotenuse is the same so we have

hypotenuse + hypotenuse + 1/2 circumference

2 sqrt(29) + 2 sqrt(29) + 4 pi

4 sqrt(29) + 4pi  cm

Step-by-step explanation:

First we need to deal with the half circle. The radius of this circle is 4, because the diameter is 8. The formula for the circumference of a circle is 2piR.

2pi4 so the perimeter for the half circle would be 8pi/2.

The area of that half circle would be piR^2 so 16pi/2.

Now moving on the triangle part, we need to find the hypotenuse side of AC. We will use the pythagoram theorem. 4^2+10^2=C^2

16+100=C^2

116=C^2

C=sqrt(116)

making the perimeter of this triangle 2×sqrt(116)

The area of this triangle is 8×10=80, than divided by 2 which is equal to 40.

We than just need to add up the perimeters and areas for both the half circle and triangle.

The area would be equal to 8pi+40

The perimeter would be equal to 4pi+4(sqrt(29))

Tim owns a clothing store where he designs pairs of shorts, s, and T-shirts, t.
He sells the shorts for $12 and the T-shirts for $8 each. Tim can work 18
hours a day, at most. It takes him 30 minutes to design a T-shirt and 45
minutes to design a pair of shorts. He must design at least 12 items each
day, but he cannot design more than 30 items in one day. Which set of
inequalities below represents this scenario?
A. s 2 12 + $ ss 30 + tiss 24-0.66t, s2; 0; t2 0
B. s> 12-tss 30 - t Ss 24 -0.66t, s 2 0; t> 0
O c. s? 12-ts? 30-tss 24 -0.66t, s 2 0;t20
D. S 2 12-tss 30-ts 24 -0.66t, s 20; t 0​

Answers

Answer:

The correct option is;

B. s ≥ 12 - t, s ≤ 30 - t, s ≤ 24 - 0.66·t

Step-by-step explanation:

The given parameters are;

The number of T-shirts, t, and shorts, s, Tim must design a day = 12

The maximum number of T-shirts and shorts Tim can design a day = 30

The maximum number of hours Tim can work = 18 hours

Therefore, we have;

The number of shorts Tim designs in a day is ≥ The minimum number of T-shirts and shorts Tim must design a day less the number of T-shirts Tim designs

Which gives;

s ≥ 12 - t

Also the number of shorts Tim designs in a day is ≤ The maximum number of T-shirts, and shorts, Tim can design a day less the number of T-shirts Tim designs

Which gives;

s ≤ 30 - t

The number of 45 minute period for the design of shorts in 18 hours = 18×60/45 = 24

The fraction of 36 minutes in 45 minutes = 36/45 = 0.667

Therefore we have;

The number of shorts Tim designs in a day is ≤ The number of 45 minute periods in 18 hours less the number of 36 minutes periods used to design T-shirts

Which gives;

s ≤ 24 - 0.66·t

The correct option is s ≥ 12 - t, s ≤ 30 - t, s ≤ 24 - 0.66·t.

Answer:

B. s> 12-tss 30 - t Ss 24 -0.66t, s 2 0; t> 0

Step-by-step explanation:

Hope this helps!!

HELP SOMEONE PLEASE!!!!! Factor completely 10x2 + 2x − 8. 2
(5x − 1)(x + 4) 2(5x − 4)(x + 1) 2(5x + 2)(x − 2) 2(5x − 2)(x + 2)

Answers

Answer:

2(5x - 4)(x + 1)

Step-by-step explanation:

10x^2 + 2x − 8 =

First, factor out the GCF of all terms which is 2.

= 2(5x^2 + x - 4)

5x^2 factors into 5x and x.

= 2(5x      )(x       )

-4 factors into -4 and 1, -1 and 4, and -2 and 2. Use the set of two factors in the proper positions that will give the middle term.

= 2(5x - 4)(x + 1)

Answer:

[tex]\large \boxed{2(5x-4)(x+1)}[/tex]

Step-by-step explanation:

[tex]10x^2 + 2x - 8[/tex]

Rewrite 2x as 10x - 8x.

[tex]10x^2 + 10x-8x - 8[/tex]

Factor out the two groups.

[tex]10x(x+1)-8(x+1)[/tex]

Take x+1 as a common factor.

[tex](10x-8)(x+1)[/tex]

Factor 10x - 8.

[tex]2(5x-4)(x+1)[/tex]

what is the coefficient of x in the equation of 32+2x=10
solve after finding the coefficient

Answers

Answer:

x= -11

Step-by-step explanation:

the coefficient is variable that appears before a number . bearing this in mind, the coefficient of x is therefore 2 .

the value of x is:

>32+2x=10

>2x=10-32

>2x= -22

>x= -11

Answer:

Step-by-step explanation:

Coefficient of x = 2

32 + 2x  = 10

Subtract 32 from both side

32 + 2x -32 = 10 - 32

              2x =  - 22

Divide both sides by 2

            2x/2 = -22/2

x = -11

What is the width of the rectangle shown below?
4x + 3
A = 8x2 – 10x – 12

Answers

Answer:

2x-4

Step-by-step explanation:

Area of a rectangle = Length * Width

Given parameters

Area A = 8x2 – 10x – 12

Length of the rectangle = 4x+3

Required

Width of the rectangle.

Substituting the given parameters into the formula

8x2 – 10x – 12  = (4x+3)*width

width = 8x2 – 10x – 12 /4x+3

S

Factorizing the numerator

8x² – 10x – 12

= 2(4x²-5x-6)

= 2(4x²-8x+3x-6)

= 2(4x(x-2)+3(x-2))

= 2(4x+3)(x-2)

Width = 2(4x+3)(x-2)/4x+3

Width = 2(x-2)

Width = 2x-4

Hence the width of the rectangle is 2x-4

Combine the radicals. 2√24+5√54 A) 53√6 B) 5√6 C) 19√6 D) 93√6

Answers

Answer:

The answer is option C

Step-by-step explanation:

2√24 + 5√54

To combine the radicals first make sure the radicals have the same square root

That's

For 2√24

[tex]2 \sqrt{24} = 2 \sqrt{4 \times 6} = 2 \times 2 \sqrt{6} [/tex][tex] = 4 \sqrt{6} [/tex]For 9√54

[tex]5 \sqrt{54} = 5 \sqrt{9 \times 6} = 5 \times \sqrt{9} \times \sqrt{6} [/tex][tex] = 5 \times 3 \times \sqrt{6} [/tex][tex] = 15 \sqrt{6} [/tex]

Since they have the same square root we can combine them

That's

[tex]4 \sqrt{6} + 15 \sqrt{6} = (4 + 15) \sqrt{6} [/tex]

We have the final answer as

[tex]19 \sqrt{6} [/tex]

Hope this helps you

Other Questions
Write a method named coinFlip that accepts as its parameter a string holding a file name, opens that file and reads its contents as a sequence of whitespace-separated tokens. Assume that the input file data represents results of sets of coin flips. A coin flip is either the letter H or T, or the word Heads or Tails, in either upper or lower case, separated by at least one space. You should read the sequence of coin flips and output to the console the number of heads and the percentage of heads in that line, rounded to the nearest whole number. If this percentage is 50% or greater, you should print a "You win!" message; otherwise, print "You lose!". For example, consider the following input file: H T H H T Tails taIlS tAILs TailS heads HEAds hEadS For the input above, your method should produce the following output: 6 heads (50%) You win! The American Revolution wasA-War of Independence fought between France and the American colonists in anattempt to break free from oppressive ruleB - was fought between the American colonists and Great Britain in an attempt to breakfree from oppressive ruleC - is an event that does not really have any impact on our livesD - none of the above One of the solutions of the system of equations shown in the graph has an x-value of -4. What is its corresponding integer y-value? -1 -3 0 3 Define, describe, compare, and contrast common-law crimes and statutory crimes. Be sure to provide examples. (4 points) Determine whether each of these functions is O(x 2 ). Proof is not required but it may be good to try to justify it (a) 100x + 1000 (b) 100x 2 + 1000 (c) x 3 100 1000x 2 (d) x log x (2) (2 points) U Question 1 (Multiple Choice Worth 3 points)(07.04)The cost of 3 slices of pizza is $4.89. What is the cost of each slice of pizza?O $1.63$1.89O $2.45O $2.88 Your boss has asked you to hand deliver five invitations to a special luncheon he is hosting. When you receive theinvitations, they have only first and last names but no addresses. You remember that they all live side by side in anapartment building on Central Street. The boss left the following information with his assistant, but it is all you have. Time toproblem-solve to figure out who lives where!This much you know for sure Carly has Greg as one next-door neighbor and the Joneses as her other next-doorneighbors. Now it's it up to you to figure out who lives in the other apartments. It may be helpful for you to draw five boxesto use as a visual guide. Furthermore, writing names on your drawing in pencil will allow you to keep track of the neighborsas you place them in their homes.1 The Smiths live in the westernmost apartment, Louis lives in the easternmost2. Leon has Mia as one next-door neighbor with TJ on the other side.3. Both Tami and TJ live east of the Williamses.4. TJ lives next door to the Browns5. Tom lives west of the Garcias and east of Carly6. Kris and Tami are next-door neighbors. The Garcias also live next to Tami but on the other side.7. Nikki lives east of TJ. What are the values and philosophy adopted in the Constitution of India? Explain.(5 marks)will mark u as brainliest if u answer correctly someone please help me Which statement(s) is/are true about the civil rights African Americans were guaranteed at the time of Dr. Kings speech? Select all that apply. the right to a living wage the right to vote in all states the right to attend integrated schools the right to be served in hotels, restaurants, and train stations Bobby's father has been prescribed an oral hypoglycemic drug. Such drugs act: 1. To reduce insulin resistance 2. To decrease the body's need for glucose in body cells 3. To prevent the formation of glucose 4. As an insulin replacement" What is the quick key to highlighting a column?Ctrl + down arrowCtrl + Shift + down arrowRight-click + down arrowCtrl + Windows + down arrow A.While the traits studied in Exercise 1 were hypothetical genetic traits, what type of genetic traits do you think are important to study and predict Please answer this question now how to write a diary entry using paragraph Write and solve an equation to answer the question. How many people must attend the third show so that the average attendance per show is 3000? Your client is 40 years old; and she wants to begin saving for retirement, with the first payment to come one year from now. She can save $5,000 per year; and you advise her to invest it in the stock market, which you expect to provide an average return of 9% in the future. HELP ME PLZZZ!! ILL MARK YOU BRAINLIEST Which statement is an ineffective claim? Fast food restaurants are dangerous to America's youth. The driving age should be raised for three clear reasons. The United States should increase spending for nuclear power. Violent video games make teenagers more violent. what is the value of this expression when g= -3.5?8-|2g-5|a. 20b. 10c. 6d. -4 Q1: You have to select an idea developing an application like web/mobile or industrial, it should be based on innovative idea, not just a simple CRUD application. After selecting the idea do the following: 1) How your project will be helpful and what problem this project addresses. (10-Marks) 2) Write down the requirements. (10Marks) 3) List the functional and non-functional requirements of your project. (10marks) 4) Which process model you will follow for this project and why? (10marks) 5) Draw the Level 0, and level 1 DFD of your application. (20marks)